Vous êtes sur la page 1sur 15

Modern Quantum Field Theory (PGPH11094)

1. Consider the real scalar field theory in d space-time dimensions with the Lagrangian
density L = L0 + LInt with
n  
X 1 1 2 2
L0 [{i }] = i i mi i .
i=1
2 2

(a) Write down the defining path integral for the generating functional in the free theory.
[2]

(b) Perform the path integral above and express the result in terms of configuration-space
propagators i (x y). [4]

(c) Consider henceforth n = 2 with the bare Lagrangian density:

b 3 gb
L[{b i }] = L0 [{b i }] + b + b 1 b 22 .
6 1 2
where the b subscript indicates bare fields and parameters. Write down the renor-
malised Lagrangian corresponding to this theory, and separate it according to L =
L0 + LInt . You may include linear terms in the field if these are needed explain your
reasoning. [3]

(d) Draw diagrams corresponding to each of the terms in LInt and write down the corre-
sponding momentum-space Feynman rule. [4]

(e) Draw connected diagrams corresponding to the following correlation functions through
one loop: [4]

(i) h0|1 (x)|0i (all contributions)


(ii) h0|T (2 (x)2 (y)) |0i (all contributions)
(iii) h0|T (1 (x)1 (y)2 (z)2 (w)) |0i (10 examples)

(f) Draw the diagrams that contribute to the renormalization factors associated with
and g at one loop order. [3]

(g) Use the Feynman rules to write down the expression for the one-loop correction to the
renormalization of g, and determine the number of space-time dimensions in which
this integral is finite, and when it is divergent. (There is no need to evaluate any
integral). [2]

(h) Write down the relation between the bare coupling constants b and gb and the cor-
responding renormalized parameters in terms of the renormalization factors of the
couplings and fields. Hence, assuming d = 6 2 dimensions, relate the scale depen-
dence of the couplings to that of the renormalization factors. [3]

Printed: Monday 20th June, 2016 Page 2 Continued overleaf. . .


Modern Quantum Field Theory (PGPH11094)

2. Consider the Kallen-Lehmann representation and the on-shell scheme self-energy function
in six-dimensional 3 theory.

(a) Consider the Fourier transform of the full propagator in a scalar field theory in d
space-time dimensions,
Z
F (p ) = dd x exp(ip x) h0| T ((x)(0)) |0i .
e 2

E
By inserting a complete set of states p with masses m and spatial momentum p,

e F (p2 ).
derive the Kallen-Lehmann representation of the propagator [5]
e F (p2 ) in a scalar QFT can be expressed as
(b) The full propagator

De F (p2 )
e F (p2 ) =
,
1 i(p2 ) De F (p2 )

where De F (p2 ) is the free propagator. Show, by comparing with the Kallen-Lehmann
representation that, in on-shell renormalization, the self energy function (p2 ) must
admit [4]
d(p2 )

2
(m ) = 0 and = 0.
dp2 p2 =m2

(c) What type of Feynman graphs contribute to (p2 )? Explain your answer. Draw the
contributing 1-loop diagram(s), and an example of a relevant two-loop graph. [2]

(d) Write the expressions for the integrals corresponding to the above one-loop diagram(s)
and determine their degree of divergence in d = 6 space-time dimensions. [2]

(e) Verify that the following result for the self-energy function

1 M2
Z  
2 2 2 2
(p ) = (p m ) + dxM ln ,
12 2 0 m2 (1 x(1 x))

with = g 2 /(4)3 and M 2 = m2 p2 x(1 x), is consistent with the on-shell renor-
malization conditions of (b). [4]

(f) Show that the


P Kallen Lehmann representation implies that the spectral density func-
tion (s) = |h0 |(0)| 0 i|2 (s m2 ) is given by the imaginary part of the propa-
gator: [3]
1 n e o
(s) = Im iF (s) .

(g) Show that the analytic structure of the propagator at one loop is consistent with the
Kallen Lehmann representation. Hence show that spectral density function at one
loop evaluates to [5]
" Z 1 1 2 2
#
2 2 2 1 1 2 + 2 14m /p 2 2
(p ) = (p m ) 2 dx(m p x(1 x)) .
(p m2 )2 2 12 12 14m2 /p2

Printed: Monday 20th June, 2016 Page 3 Continued overleaf. . .


Modern Quantum Field Theory (PGPH11094)

3. Consider a scalar field and a Dirac field (x) (where is a spinor index) coupled
through the following Lagrangian:
1 n l
L = i + + n + gk,l k ,
2
in d space-time dimensions, where n, k and l are all positive integers. Recall that =
0 .

(a) What are the dimensions of the fields and ? [2]

(b) What are the dimensions of the coupling constants n and gk,l appearing in the
interaction Lagrangian? [3]

(c) Which values of n and (k, l) are allowed in a renormalizable (or super-renormalizable)
theory? Refer explicitly to dimensions d = 2, d = 3 and d = 4 and d 5, and in each
case write the most general renormalizable Lagrangian density. [6]

(d) Write down the Feynman rules for the fermion and scalar propagators and the g1,1
and g2,1 interactions in momentum space. Draw the diagrams corresponding to these
two vertices using a wavy line for the scalar and a full line for the fermion. [2]

(e) Consider the fermion-fermion 2 2 scattering process with momentum assignment


p1 p2 p3 p4 , and draw one-particle irreducible one-loop diagrams involving exclu-
sively the g1,1 and g2,1 interactions, proportional to
4
(i) g1,1 ,
2
(ii) g1,1 g2,1 ,
2
(iii) g2,1 .

In each case use the Feynman rules to write down the corresponding Feynman integral
(no need to evaluate it) in d space-time dimensions. [6]

(f) For each of the three diagrams in e) determine in what number of space time dimen-
sions the integral is UV finite. [3]

(g) Draw a one-loop diagram that renormalises the vertex g2,1 . For what number of
space-time dimensions does the corresponding integral converge? [3]

Printed: Monday 20th June, 2016 Page 4 End of Paper


Modern Quantum Field Theory (PHYS11047)

1. Consider the real scalar field theory with 4 interaction in 4 space-time dimensions with
the renormalized Lagrangian density:
1 1 1
L = Z Zm m2 2 Z 4 .
2 2 4!

(a) Write down the path integral for the generating functional Z0 [J] of the free theory,
and show that in momentum space it may be expressed as: [3]

d4 p  2
Z  Z 
1 2

Z0 [J] = D exp i e (p) p + m (p)
e Je (p)(p)
e e (p)J(p)
e
2 (2)4

(b) Perform the path integral to derive an explicit result for Z0 [J]. Obtain the Feynman
propagator DF (x y) in the free theory (the answer should be expressed as a Fourier
transform). [3]

(c) Show how the generating functional for the interacting theory, Z[J], can be con-
structed perturbatively by acting on the free-theory one. Write down an all-order
expansion in the coupling (make sure that all renormalization factors are taken into
account). [5]

(d) Draw connected Feynman diagrams corresponding to tree-level and one-loop contri-
butions to correlation functions involving up to six fields (one one-loop example for
each correlation function is sufficient). For the one loop diagrams indicate in each
case whether the diagram is ultraviolet divergent, and if so indicate how you expect
it to depend on an ultraviolet momentum cutoff UV . [5]

(e) Draw all one-particle irreducible Feynman diagrams contributing to the vertex correc-
tion i(p1 , p2 , p3 , p4 ) through one loop, and write down the corresponding expressions
according to the Feynman rules. [3]

(f) Compute the one-loop contribution to i(p1 , p2 , p3 , p4 ) in dimensional regularization


with d = 4 2 dimensions using Feynman parametrization. Evaluate the momentum
integral and find for what values of momenta the integral has an imaginary part.
Note: there is no need to evaluate the Feynman parameter integral. [6]

Printed: Friday 17th April, 2015 Page 2 Continued overleaf. . .


Modern Quantum Field Theory (PHYS11047)

2. Consider spinor electrodynamics described by the following renormalized Lagrangian:


1 ( A )2
L = (Z2 i/ + Z1 eA/) Zm m Z3 F F ,
4 2

where the free Feynman propagator, D e (p) is given by


 
i p p p p
De (p) =
2
T (p) + 2 , where T = g 2 .
p + i p p

(a) Show that the exact momentum-space photon propagator


e (p) may written as


e (p) = D e (p)i (p)D
e (p) + D e (p) + . . . ,

and characterize the class of diagrams that contribute to i (p). [2]

(b) Prove that T acts as a transverse projection operator, i.e. that p T = 0 and that
T T = T . [3]

(c) Assuming that (p) is transverse, i.e. that (p) = p2 (p2 )T , sum up the series
in (a). [4]

(d) Draw the diagram contributing to i (p) at one-loop order and write down the
corresponding momentum-space expression in d space-time dimensions. [3]

(e) Compute the following integral over the Minkowski-space loop momentum l and show
that
dd l l l 1 (n d2 1)
Z
1
 d +1n
d 2 2 n
= ig d/2
M 2 i 2 .
(2) (l + M i) 2 (4) (n)

You may use equation (3) of the useful formulae in the beginning of the exam paper.
[3]

(f) Assuming that the diagram in (d) yields the following integral
Z 1 Z
bubble 2 dd l 8p p x(1 x) 4g (m2 l2 + x(1 x)p2 ) 8l l
i (p) = e dx 2 ,
(2)d

0 l2 + m2 x(1 x)p2 i

evaluate the momentum integral and show that it yields a transverse Lorentz tensor
(i.e. show that it is proportional to T (p)). [7]
Guidance:
Note that you are not expected to evaluate the Feynman parameter integral.
Recall that x(x) = (1 + x).

(g) Find the renormalization constant Z3 at one-loop order in a minimal subtraction


scheme. [3]

Printed: Friday 17th April, 2015 Page 3 Continued overleaf. . .


Modern Quantum Field Theory (PHYS11047)

3. Consider Yukawa theory containing a Dirac fermion , and a real pseudo-scalar field ,
with the bare Lagrangian density
1 1 0
L = i0 /0 m0 0 0 + 0 0 M02 20 40 + ig0 0 0 5 0 .
2 2 4!
(a) Write down the renormalized Lagrangian density, determine the dimension of the
fields and and coupling constants and g in d = 4 2 space-time dimensions. [4]
(b) Working in dimensional regularization with d = 4 2 space-time dimensions, show
that the relations between the bare and renormalized couplings are
g0 = Z1/2 Z1 Zg
 g , 0 = Z2 Z
2 ,
where
is introduced to render g and dimensionless. [4]
(c) Draw the one-loop Feynman diagrams that need to be computed to determine each
of the renormalization constants given below. Identify which diagram contributes to
each term:
g2 1
 
Z = 1 2 + finite
8 
g 2 m2
  
1
ZM = 1 + + finite
32 2 4 2 M 2 
g2
 
1
Z = 1 + finite
32 2 
g2
 
1
Zm = 1 + finite
16 2 
g2
 
1
Zg = 1 + + finite
16 2 
3g 4
  
3 1 [6]
Z = 1 + + finite .
32 2 2 2 

(d) Write down the expression corresponding to the diagram associated with Zg . Explain
why it is expected to yield a O(1/) pole upon performing the loop momentum integral
(there is no need to evaluate the integral). [4]
(e) Working in the (modified) minimal subtraction scheme, define

 X Gn (g, )
ln Z1/2 Z1 Zg =
n=1
n
and compute the one-loop contributions to G1 (g, ) using the results of the renormal-
ization factors given above. [3]
(f) Show that the function, defined by
dg
= g + g (g, ) ,
d ln
where g (g, ) is independent of , is given by
 

g (g, ) = g g + 2 G1 ,
g
Use these relations to determine the one-loop beta function in terms of g and . [4]

Printed: Friday 17th April, 2015 Page 4 End of Paper


Modern Quantum Field Theory (PHYS11047)

1. Consider the n scalar field theory in d space-time dimensions with the renormalized
Lagrangian density:
1 1 1
L = Z Zm m2 2 Zg gn n .
2 2 n!

(a) Determine the mass dimension of the field and the coupling constant gn . Find
the values of n for which the theory is renormalizable for d =3, 4 and 6. [5]

(b) From now on consider d = 6 and n = 3. Draw the (one-particle irreducible) one-
loop Feynman diagrams that contribute to the 4-point function iV4 (p1 , p2 , p3 , p4 ).
Use the Feynman rules to write the integral for one such diagram, ib(p1 , p2 , p3 , p4 ),
in terms of qi = ij=1 pj .
P
[4]

(c) By power counting show that the integral you got in (b) is ultraviolet finite, and
explain why this fact is essential for the renormalizability of the theory. [2]

(d) Draw an ultraviolet-divergent one-particle-irreducible two-loop Feynman diagram


contributing to the 4-point function iV4 (p1 , p2 , p3 , p4 ), write down the corresponding
expression, and show by power counting it is indeed divergent. Explain whether
this can be consistent with renormalizability. [7]

(e) Consider the integral you obtained in (b) above and show that

g 4 Z 1 Z 1x Z 1xy 1
b(p1 , p2 , p3 , p4 ) = 3
dx dy dz 2
(4) 0 0 0 Mb

and find Mb 2 as a function of qi , m and the integration parameters x, y and z. [7]

Printed: Thursday 17th April, 2014 Page 2 Continued overleaf. . .


Modern Quantum Field Theory (PHYS11047)

2. Consider Quantum Electrodynamics

(a) The free Lagrangian density of an electron and a photon (in a covariant gauge) is
given by
1 ( A )2
L0 = i/ m F F .
4 2
Write down the renormalized Lagrangian density in the interacting theory, with the
renormalization factors as follows: Z1 for the interaction term, Z2 for the electron
wave function, Zm for the electron mass, and Z3 for the photon wave function. [4]

(b) The Kallen-Lehmann representation of the electron propagator in QED reads


Z
i i (s)
e /)
S(p = + 2 ds
p/ m + i mth p/ s + i

where m is the electron mass and p is its 4-momentum.


What is the value of the m2th ? Which states contribute to the spectral density
function (s) near the lower limit of the integral? [3]

(c) i(p/), the electron self-energy function, is defined as the 1-particle irreducible two-
point function. Draw one-, two- and three-loop diagrams which contribute to i(p/)
(one example of each would suffice). Draw also a three-loop diagram that contributes
to the electron propagator S(pe /), but not to i(p/). [4]

(d) Derive the expression for the electron propagator in terms of the self energy and
then determine the on shell renormalization conditions. [7]

(e) Assume that the one-loop diagram contributing to (p/), where the photon propa-
gator is modified to include a mass m , yields
" #
p/ + 4m Z 1  
+ p/ 2m dx 2(1 x)p/ + 4m ln M 2 /2
4  0

with M 2 = p2 x(1 x) + xm2 + (1 x)m2 . Find the renormalization factors Zm


[7]
and Z2 in the on-shell scheme to one loop accuracy. Explain why m is introduced.

Printed: Thursday 17th April, 2014 Page 3 Continued overleaf. . .


Modern Quantum Field Theory (PHYS11047)

3. Consider Yukawa theory containing a Dirac fermion , and a real scalar field , with the
bare Lagrangian density
1 1 0
L = i0 /0 m0 0 0 + 0 0 M02 20 40 g0 0 0 0 .
2 2 4!

(a) Write down the renormalized Lagrangian density, determine the dimension of the
fields and and coupling constants and g in d = 4 2 space-time dimensions,
and explain why the theory is expected to be renormalizable in 4 dimensions. [4]
(b) Working in dimensional regularization with d = 4 2 space-time dimensions, show
that the relation between the bare and renormalized couplings are
g0 = Z1/2 Z1 Zg
 g , 0 = Z2 Z
2 ,
where
is introduced to render g and dimensionless. [4]
(c) Draw the one-loop Feynman diagrams that need to be computed to determine each
of the renormalization constants given below. Identify which diagram contributes
to each term:
g2 1
 
Z = 1 + finite
8 2 
g 2 m2
!
1

ZM = 1 + + finite
32 2 4 2 M 2 
g2 1
 
Z = 1 + finite
32 2 
g2 1
 
Zm = 1 + finite
16 2 
g2 1
 
Zg = 1 + + finite
16 2 
3g 4
!
3 1

Z = 1 + + finite [6]
32 2 2 2 

(d) Working in the (modified) minimal subtraction scheme, define



  Gn (g, )   Ln (g, )
ln Z1/2 Z1 Zg = ln Z2 Z =
X X

n=1 n n=1 n
and compute the one-loop contributions to G1 (g, ) and L1 (g, ) using the results
of the renormalization factors given above. [4]
(e) Show that the functions, defined by
dg d
= g + g (g, ) , = 2 + (g, ) ,
d ln d ln
where g (g, ) and (g, ) are independent of , are given by
! !

g (g, ) = g g + 2 G1 , (g, ) = g + 2 L1 .
g g
Use these relations to determine the one-loop beta functions in terms of g and . [7]

Printed: Thursday 17th April, 2014 Page 4 End of Paper


Modern Quantum Field Theory (PHYS11047)

1. The full propagator


e F (p2 ) in a scalar QFT can be expressed as

De F (p2 )
e F (p2 ) =
,
1 i(p2 ) De F (p2 )

e F (p2 ) is the free propagator.


where D

(a) What type of Feynman graphs contribute to (p2 )? Explain your answer
and draw all the relevant 1- and 2-loop diagrams in 3 and 4 theories (you
may discard diagrams with counter terms). [6]

(b) Write the expressions for the integrals corresponding to the above one-loop
diagrams and determine their degree of divergence in d = 4 space-time
dimensions. [4]

(c) Determine the on-shell renormalization conditions for (p2 ) consistent with
the Kallen Lehmann representation [3]
Z
2 i i
F (p ) = 2
e
2
+ ds (s) 2 .
p m + i 4m2 p s + i

(d) Given the one-loop result for (p2 ) in the 3 theory in dimensional regu-
larization with d = 6 2 dimensions,

( 1) 1 M2
Z 
(1) 2 2
(p ) = dxM 2
+ Z(1) p2 Zm
(1) 2
m ,
2 0 e E

where M 2 = m2 x(1 x)p2 i, determine the field and mass one-loop


(1) (1)
counter terms Z and Zm in the on-shell scheme (expressed as parametric
integrals over x). Deduce that the renormalized (1) (p2 ) is given by

1 1 M2
Z  
(1) 2 1 2 2 2
(p ) = (p m ) + dxM ln .
12 2 0 m2 (1 x(1 x))
[7]

(e) Show that the analytic structure of the propagator at one loop is consistent
with the Kallen Lehmann representation. [5]

Printed: May 5, 2014 Page 1 Continued overleaf. . .


Modern Quantum Field Theory (PHYS11047)

2. Consider the k scalar field theory with the renormalized Lagrangian density:
1 1 1
L = Z Zm m2 2 + Zg gk .
2 2 k!

(a) Determine the mass dimension of the field and the coupling constant g in
d space-time dimensions. [2]

(b) What is the degree of divergence of a diagram with E external legs and
vk vertices of the k type? Determine what values of k are allowed in a
renormalizable theory in 4 space-time dimensions. [6]

(c) Write down the bare Lagrangian density corresponding to L above, and
determine the relation between the bare field, mass and coupling and the
renormalized ones. [3]

(d) Specialising now to the 3 theory in d = 6 2 dimensions find the relation


between the bare coupling 0 = g02 /(4)3 , its renormalized counterpart
(
), and the scale , and use it to derive the expression for the function,
() = d/d in terms of Z () and Zg (). [7]

(e) Given the one loop renormalization constants for the 3 theory for d = 62
dimensions in the MS scheme
1 1 1
Z = 1 + O(2 ) , Zg = 1 + O(2 ) . (1)
12  2
determine the function through O(2 ). Sketch the form of the beta
function as a function of the coupling (A) for  = 0 (B) for  < 0 and
comment on the physical consequences. [7]

Printed: May 5, 2014 Page 2 Continued overleaf. . .


Modern Quantum Field Theory (PHYS11047)

3. Consider Maxwell theory in 4 dimensions.

(a) Write down the expression for the Lagrangian density L and show that it
is invariant under infinitesimal local gauge transformations [2]
1
A (x) A (x) (x).
e

(b) Starting from the expression for L show that the action can be written as

d4 k e
Z
1
S= 4
A (k)(k 2 )T A
e (k) ,
2 (2)

and determine the tensor T . [5]

(c) Show that longitudinal modes do not contribute to the action. [2]

(d) Follow the Faddeev Popov procedure to derive the generating functional for
the Maxwell theory with a source term under a gauge condition G(A) = 0.
[8]

(e) Use your result to derive the photon propagator in a covariant gauge where
G(A) = A w(x).
Hint: introduce a gauge parameter by integrating over w(x) with a Gaus-
sian weight of width . [8]

Printed: May 5, 2014 Page 3 End of paper


Modern Quantum Field Theory (PHYS11047)

1. A four-dimensional massive real scalar theory is described by the following La-


grangian density
1 1
L = m2 2 n .
2 2 n!

(a) Consider the free theory where = 0. Include an external source, linear in
, write down the action in coordinate and in momentum space and then
perform a functional integral to determine the generating functional for the
free theory as a function of the source. [4]

(b) Use the result in (a) to determine the free propagator. [2]

(c) Write down the expression for the generating functional in the interacting
theory using functional derivatives with respect to the source. Remember
to include the necessary counter-terms. [3]

(d) Explain how one can impose the requirement that h0| (x) |0i should vanish.
What is the diagrammatic interpretation of this requirement. Distinguish
between odd and even n. [3]

(e) Ignoring counterterms, what vacuum diagrams are there for n = 4 at


O(2 )? Draw the diagrams and write down the corresponding expressions
as integrals in configuration space involving the free propagator. [6]
Remember to include symmetry factors (there is no need to explain how
you got them).

(f) For n = 4, draw a one-loop diagram with two vertices, write down the mo-
mentum space integral, determine whether it is ultraviolet finite and in case
it is divergent whether this divergence may be removed by renormalization.
Repeat the exercise for n = 6. [7]

Printed: August 13, 2012 Page 2 Continued overleaf. . .


Modern Quantum Field Theory (PHYS11047)

2. Consider a massive real scalar theory with mass m and an interaction term
gZg 3
LI =
3!
in dimensional regularization with d = 6 2 dimensions, for small positive .

e (where
(a) Show that the rescaling g = ge e carries mass dimension one)
renders ge dimensionless. [5]

(b) Draw the (one-particle irreducible) one-loop diagram contributing to the


3-point vertex function and write down the corresponding expression in
momentum space, assuming all three momenta p1 , p2 and p3 are defined as
incoming. [5]

(c) Show that the one-loop momentum integral in (b) yields


Z 1 Z 1x Z
triangle 3 dd k 1
iV (p1 , p2 , p3 ) = g (3) dx dy . (6)
0 0 (2) (k + M 2 )3
d 2

Determine the scale M 2 as a function of m2 , p21 , p22 and p1 p2 as well as


the Feynman parameters x and y. [7]
Guidance: Eq. (1) may be used. Note also that Zg may be set to 1.

(d) Show that if the renormalization factor is set as


 
1 1
Zg = 1 + + ln(m / ) + O(2 )
2 2
2 2

where 2 4e 2 eE and = ge2 /(4)3 while is an arbitrary finite


constant, then one gets the following expression for the renormalized vertex,
 Z 1 Z 1x  2 
M
V (p1 , p2 , p3 )/g = 1 + dx dy ln 2
+ O(2 )
0 0 m

where finite terms for  0 have been dropped. [8]


Guidance:
Recall that () = 1/ E + O() .
For d-dimensional Euclidean integrals you may use eq. (2).

Printed: August 13, 2012 Page 3 Continued overleaf. . .


Modern Quantum Field Theory (PHYS11047)

3. Consider spinor electrodynamics described by the following (bare) Lagrangian:


1 ( A )2
L = (i/ + eA/) m F F ,
4 2
and assume that the exact momentum-space photon propagator is written as a
geometrical series of the form


e (p) = D e (p)i (p)D
e (p) + D e (p) + . . . , (7)

where D
e (p) is the free Feynman propagator,
 
i p p p p
D (p) = 2
e T (p) + 2 , where T = g 2 ,
p + i p p

and where i (p) is the sum of all one-particle irreducible Feynman diagrams
with two external photon lines (not including the external propagators).

(a) Prove that T acts as a transverse projection operator, i.e. that p T = 0


and that T T = T . [2]

(b) Assuming that (p) is transverse, i.e. that (p) = p2 (p2 )T , sum
up the geometrical series in (7). [5]

(c) Draw the one-particle irreducible diagram contributing to i (p) at one-


loop order and write down the corresponding momentum-space expression
in d space-time dimensions. [3]
i(q/ + m)
Recall that the free fermion propagator is S(q) = 2 .
q m2 + i
(d) Show that the diagram in (c) yields the following integral
Z 1 Z
bubble 2 dd l 8p p x(1 x) 4g (m2 l2 + x(1 x)p2 ) 8l l
i (p) = e dx 2
(2)d

0 2 2 2
l + m x(1 x)p i

Guidance: to perform the trace over spinor indices you may use eqs. (4). [7]

(e) Evaluate the momentum integral in (d) and show that it yields a transverse
Lorentz tensor (i.e. show that it is proportional to T (p)). [8]
Guidance:
Note that you are not expected to evaluate the Feynman parameter integral.
You may use eq. (5) as well as equations (2) and (3). Also recall that
x(x) = (1 + x).

Printed: August 13, 2012 Page 4 End of paper

Vous aimerez peut-être aussi